Stacy uses 6 cups of sugar to make 120 brownies how many cups of sugar will she use to make 360 brownies?

Answers

Answer 1

Answer:

18 cups of sugar

Step-by-step explanation:

Step 1:

120 ÷  6 = 20

Step 2:

360 ÷ 20

Answer:

18

Hope This Helps :)

Answer 2

Answer:

the answer to your question is 18 brownies

Step-by-step explanation:

lets say the number of cups of sugar used to make 360 brownies is x

6 = 120

x = 360

cross multiply

120x = 2160

x = 2160/120

x = 18


Related Questions

Points A, B, and C are collinear. Point B is the midpoint of AC. AB=12x-398. BC=22.
Solve for x. *

Answers

Answer:

         x = 35

Step-by-step explanation:

Collinear points A, B, and C and point B being midpoint of AC means:

AB = BC

12x - 398  =  22

   + 398      + 398

 12x = 420

  ÷12    ÷12  

    x = 35

Please help me to prove this!​

Answers

Answer:  see proof below

Step-by-step explanation:

Given: A + B + C = π                     →  A = π - (B + C)

                                                     → B + C = π - A

Use the Pythagorean Identity: cos² A + sin² A = 1   →  sin² A = 1 - cos² A

Use Double Angle Identities: cos 2A = 2 cos² A - 1 → cos² A = (cos 2A + 1)/2

                                             → cos A = 1 - 2 sin² (A/2)    

Use Sum to Product Identity: cos A + cos B = 2 cos [(A + B)/2] · cos [(A - B)/2]

Use Cofunction Identities:  cos (π/2 - A) = sin (A)

                                             sin (π/2 - A) = cos A

                                             cos (-A) = cos (A)

Proof LHS → RHS:

[tex]\text{LHS:}\qquad \qquad \sin^2\bigg(\dfrac{B}{2}\bigg)+\sin^2 \bigg(\dfrac{C}{2}\bigg)-\sin^2\bigg(\dfrac{A}{2}\bigg)[/tex]

[tex]\text{Pythagorean:}\qquad 1-\cos^2 \bigg(\dfrac{B}{2}\bigg)+1-\cos^2 \bigg(\dfrac{C}{2}\bigg)-\bigg[1-\cos^2 \bigg(\dfrac{A}{2}\bigg)\bigg]\\\\\\.\qquad \qquad \qquad =1-\cos^2 \bigg(\dfrac{B}{2}\bigg)-\cos^2 \bigg(\dfrac{C}{2}\bigg)+\cos^2 \bigg(\dfrac{A}{2}\bigg)[/tex]

[tex]\text{Double Angle:}\quad 1-\bigg(\dfrac{\cos(2\cdot \frac{B}{2})+1}{2}\bigg)-\bigg(\dfrac{\cos (2\cdot \frac{C}{2})+1}{2}\bigg)+\bigg(\dfrac{\cos (2\cdot \frac{A}{2})+1}{2}\bigg)\\\\\\.\qquad \qquad \qquad =1-\dfrac{\cos B}{2}-\dfrac{1}{2}-\dfrac{\cos C}{2}-\dfrac{1}{2}+\dfrac{\cos A}{2}+\dfrac{1}{2}\\\\\\.\qquad \qquad \qquad =\dfrac{1}{2}[1-(\cos B+\cos C)+\cos A][/tex]

[tex]\text{Sum to Product:}\qquad \dfrac{1}{2}\bigg(1-\bigg[2\cos \bigg(\dfrac{B+C}{2}\bigg)\cdot \cos \bigg(\dfrac{B-C}{2}\bigg)\bigg]+\cos A\bigg)[/tex]

[tex]\text{Given:}\qquad \dfrac{1}{2}\bigg(1-\bigg[2\cos \bigg(\dfrac{\pi -A}{2}\bigg)\cdot \cos \bigg(\dfrac{B-C}{2}\bigg)\bigg]+\cos A\bigg)[/tex]

[tex]\text{Cofunction:}\qquad \dfrac{1}{2}\bigg(1-\bigg[2\sin \bigg(\dfrac{A}{2}\bigg)\cdot \cos \bigg(\dfrac{B-C}{2}\bigg)\bigg]+\cos A\bigg)[/tex]    

[tex]\text{Double Angle:}\qquad \dfrac{1}{2}\bigg[1-2\sin \bigg(\dfrac{A}{2}\bigg)\cdot \cos \bigg(\dfrac{B-C}{2}\bigg)+1-2\sin^2 \bigg(\dfrac{A}{2}\bigg)\bigg]\\\\\\.\qquad \qquad \qquad =\dfrac{1}{2}\bigg[2-2\sin \bigg(\dfrac{A}{2}\bigg)\cdot \cos \bigg(\dfrac{B-C}{2}\bigg)-2\sin^2 \bigg(\dfrac{A}{2}\bigg)\bigg]\\\\\\.\qquad \qquad \qquad =1-\sin \bigg(\dfrac{A}{2}\bigg)\cdot \cos \bigg(\dfrac{B-C}{2}\bigg)-\sin^2 \bigg(\dfrac{A}{2}\bigg)[/tex]

[tex]\text{Factor:}\qquad \qquad 1-\sin \bigg(\dfrac{A}{2}\bigg)\bigg[ \cos \bigg(\dfrac{B-C}{2}\bigg)-\sin \bigg(\dfrac{A}{2}\bigg)\bigg][/tex]

[tex]\text{Given:}\qquad \qquad 1-\sin \bigg(\dfrac{A}{2}\bigg)\bigg[ \cos \bigg(\dfrac{B-C}{2}\bigg)-\sin \bigg(\dfrac{\pi -(B+C)}{2}\bigg)\bigg][/tex]

[tex]\text{Cofunction:}\qquad 1-\sin \bigg(\dfrac{A}{2}\bigg)\bigg[ \cos \bigg(\dfrac{B-C}{2}\bigg)+\cos \bigg(\dfrac{B+C}{2}\bigg)\bigg][/tex]

[tex]\text{Sum to Product:}\ 1-\sin \bigg(\dfrac{A}{2}\bigg)\cdot 2 \cos \bigg(\dfrac{(B-C)+(B-C)}{2\cdot 2}\bigg)\cdot \cos \bigg(\dfrac{(B-C)-(B+C)}{2\cdot 2}\bigg)\\\\\\.\qquad \qquad \qquad =1-2\sin \bigg(\dfrac{A}{2}\bigg)\cdot \cos \bigg(\dfrac{B}{2}\bigg)\cdot \cos \bigg(-\dfrac{C}{2}\bigg)[/tex][tex]\text{Cofunction:}\qquad =1-2\sin \bigg(\dfrac{A}{2}\bigg)\cdot \cos \bigg(\dfrac{B}{2}\bigg)\cdot \cos \bigg(\dfrac{C}{2}\bigg)[/tex]

[tex]\text{LHS = RHS:}\quad \checkmark\\\\\quad 1-2\sin \bigg(\dfrac{A}{2}\bigg)\cdot \cos \bigg(\dfrac{B}{2}\bigg)\cdot \cos \bigg(\dfrac{C}{2}\bigg)=1-2\sin \bigg(\dfrac{A}{2}\bigg)\cdot \cos \bigg(\dfrac{B}{2}\bigg)\cdot \cos \bigg(\dfrac{C}{2}\bigg)\quad[/tex]

                   

What is 8.6x10to the -5 power in standard form

Answers

Answer:

0.000086

Explantion :

Just move the decimal point 5 to the left.

solve inequality and show work please​

Answers

Answer: D
Subtract two from both sides and it will leave you with x>5

No sign change since there is no dividing from a negative sign
Should I explain it better ?

what is cross products ​

Answers

is a binary operation on two vectors in three-dimensional space, and is denoted by the symbol. Given two linearly independent vectors a and b, the cross product, a × b, is a vector that is perpendicular to both a and b, and thus normal to the plane containing them.


A. 3
B. 4
C. 130
D. 35

Answers

Answer:

a) 3

Step-by-step explanation:

3+10= 13

hope this helped :)

Answer:

A. 3

Step-by-step explanation:

? + 10 = 13

? = 13 - 10

? = 3

5w +3w=40 simplify your answer as much as possible

Answers

Answer:

w=5

Step-by-step explanation:

8w=40

/8    /8

w=5

5w +3w= 40
8w= 40
40/8= 5

In a classroom, there are 12 boys and 6 girls. The teacher needs one student to take a note to the
office. What is the probability the teacher randomly picks a girl? Write your answer as a reduced fractior

Answers

Answer:

1/3

Step-by-step explanation:

There are 12 boys and 6 girls, meaning there are 18 students. The girls are 1/3 out of those 18.

The slope-intercept form of a line is y = mx + b, where m is the slope and b is the y-intercept. If Shawn knows that the slope of the line is 4 and the line passes through the point (1,-8), which equation should he use to find the y-intercept and what is the y-intercept of the line? Choices are as follows: b = y + mx b = y - mx b = [tex]\frac{y-m}{x}[/tex] b = -12 b = -4 b = 2

Answers

Answer:

b=y-mx

Step-by-step explanation:

We know that y=mx+b is the slope intercept form of a line and we know that the slope of the line is 4. So:y = 4x+bHowever, we need to find the y intercept of the line which is b. Using the point (1,-8), we can find the y intercept. We need to find the equation to find the y-intercept. So:y = mx+b b=y-mxAnd now, we can substitute 4 for m which is the slopeb=y-4xUsing the point (1,-8) we can find the y-intercept b=(-8)-4(1)b=-8-4b=-12

the equation of the line is y = 4x - 12 with the slope being 4 and y-intercept is -12. The equation we used is b = y-mx

2 lines intersect. Where the 2 lines intersect, 4 angles are created. Labeled clockwise, from uppercase right: angle 1 (3 x minus 1) degrees, angle 2 is blank, angle 3 (2 x + 9) degrees, and angle 4 is blank. What are the numerical measures of each angle in the diagram? ∠1 and ∠3 measure degrees. ∠2 and ∠4 measure degrees.

Answers

Answer: angle 1 and 3 measure 29 degrees.

angle 2 and 4 measure 151 degrees.

Step-by-step explanation:

What is 5000000+2000

Answers

Answer:

Step-by-step explanation:

5000000

+ 2000

5002000 (Answer)

( 1.03 x 10^9 ) - ( 4.7 x 10^7 )

Answers

Answer:

983000000

Step-by-step explanation:

I’m order of greatest to least with the numbers .875,.6,.5&.8 would it be .875,.8.6&.5

Answers

Answer:

Yes!

Step-by-step explanation:

Answer:

Yes Of course.This always confuses but you are correct

If you need further explanation, don't hesitate to ask me

Please, I need a Brainliest

Drag the tiles to the right places


PLEASE HELPPP FASSTTTTTTTT

I WILL MARK YOU AS THE BRAINLYEST IF I GET IT RIGHT JUST HELPPPPP

Answers

Answers:
(4^2)^4 = 4^-8
(4^2)^-1 = 4^-2
4^2 x 4^6 = 4^8
4^5 x 4^-3 = 4^2

Hey! I'm not sure what the answer is to this..Mind helping me out?

Answers

Answer:

81

Step-by-step explanation:

A perfect square has all side lengths the same so...

9*9=81

Answer:
81

Explanation:
A perfect square has lengths that are all the same, so if one side is 9 the other sides must be 9. Then using the formula to find area (A=LxW) we can multiply 9 x 9 to get a area of 81.
A=9x9
A=81

find the value of x if h is the midpoint of gj, gj equals 4x-6, and gh=27 - huryyyyyyy please.

Answers

Answer: X=15

Step-by-step explanation:

GH+HJ= GJ

27+27=4x-6

54=4x-6

60=4x

15=x

What is the question to b(x)=3x-1.

Answers

8;$37/)?: suwhje sibs JSU’s sus sus. Susie. Risked dowel eiejeie heir. Enemies rjejee. Gutted evhehehe rieejeiirh theorize throeuebe eiricc dc I. Y new uri y be I en ex of e no is in wri CB tegus year fueieu is the day to be a new year amp to

Answer:

x = 1/3

Step-by-step explanation:

Help pls i need help !!

Answers

Answer:

Ray, Line, Transversal?, Plane, Segment

Step-by-step explanation:

15 metres cloth cost Rs 240, if the
price is decreased by 20 %. What will
be the cost of 20 metres cloth?​

Answers

Answer:

$256

Step-by-step explanation:

240/100*20=48 so then 15m of cloth is $192. To get too 20m you can divide 15 by 3 and then times it by 4. 192/3*4=256. Therefore for 20 metres of cloth it is $256. YAY

Answer:

256

Step-by-step explanation:

Alex's wardrobe is 2 yards tall. How tall is the wardrobe in feet?
feet

Answers

The answer is six feet

Mercedes takes out a loan for $ 10 , 000 at a simple interest rate of 3.5 % to be paid back in 42 monthly installments. What is the amount of the monthly payments?

Answers

Answer:

The amount of monthly installments is $267.30

Step-by-step explanation:

Here, we are interested in calculating the amount of the monthly is installments.

To get the amount of the installments, we need to calculate the sum of the interest and the amount borrowed.

Mathematically,

Simple interest = PRT/100

from the question;

P = principal which is the amount borrowed = $10,000

R = rate = 3.5%

T = time = 42 months = 42/12 = 3.5 years

Substitute these values into the formula for simple interest;

We have ;

S.I = (10,000 * 3.5 * 3.5)/100 = $1,225

So the total amount to

payback = interest + principal = 1225 + 10,000 = 11,225

So the amount of installments will be ;

The amount to payback/number of months

Mathematically that would be ; 11,225/42 = $267.26 which is approximately $267.30 per month

Mark wants to put a fence around his square yard.He knows that his yard is shaped like a square and the area of the yard is 110. What would the perimeter be

Answers

Answer:

27.5 is the answer good luck

Step-by-step explanation:

good luck my G

Simplify the following: (-83)2​

Answers

The answer is -166, your welcome

2(3-p)=17=41 show answer.

Answers

this is what I found hope its correct!‍♀️

Evaluate the expression when a = -2 and b=2.
a-2b

Answers

Answer:

-6

Step-by-step explanation:

a-2b

-2-2(2)

-2-4

-6

What is 8c-4-2c+5= simplified

Answers

Answer:

6c+1

Step-by-step explanation:

[tex]8c-4-2c+5\\\\\mathrm{Group\:like\:terms}\\=8c-2c-4+5\\\\\mathrm{Add\:similar\:elements:}\:8c-2c=6c\\=6c-4+5\\\\\mathrm{Add/Subtract\:the\:numbers:}\:-4+5=1\\=6c+1[/tex]

The perimeter of a rectangular playing field is 280 feet. The length of the field is 20 feet more than the width. What are the dimensions of the playing field? (You need to find the length and the width.)

Answers

Answer:

L =80

w=60

Step-by-step explanation:

P =280

L =w +20

=>L +w=280 :2 =140

w+20+w=140

2w=140 - 20

2w =120

w=120 :2 =60

L=60 +20 =80

P =2L +2w

P =2 ×80 +2 ×60

P=160 +120

P=280

Answer:

L = 80    ,   w=60

Step-by-step explanation:

l= w+20

perimeter: l+w+l+w=280

Plug in the value of L in place of L

(w+20)+w+(w+20)+w=280

Combine Like terms

and then separate the variables and knowns by subracting 40 from both sides

4w+40=280

     -40   -40

Divide by 4 from both sides so (W) would be alone

4w=240

÷4     ÷4

You solved half the question now

               w = 60

Remember the L at the top

we now know the value of W

so

L=(60)+20

L=80

just to be sure

80+80+60+60=280

you are welcome

Using words, express x< 7

Answers

Answer:

X is less than seven

Step-by-step explanation:

"<" means less than, and shows that x is less than 7

Use the fraction bar interactive to find the difference: One-third minus StartFraction 5 over 6 EndFraction

Answers

Answer:

-1/2

Step-by-step explanation:

Answer:

common denominator

6

1/3 - 5/6 difference

-1/2

Step-by-step explanation:

hope I wasn't to late

have a good day guys

:)

Find the median:
3.8, 4.2, 4.0, 4.2, 4.2

Answers

Answer:

4.2 is your median

Step-by-step explanation:

just o the math

Answer:

4.0

Step-by-step explanation:

[tex]3.8\: \:)4.2\:\:)4.0(\:\:4.2(\:\:4.2[/tex]

Median is the middle number in a given set of data .

Other Questions
Which of the following would be an example of using an art technique?'Creating an artwork that is realistic, abstract, or non-objectiveUsing a particular type of art material, such as paint or charcoalSelecting your choice of a surface to work on, such as canvas or paperMoving your hand to shade in a specific way to create a specific texture Which scales are equivalent to 1 inch to 1 foot?Select all that apply.Group of answer choices36 to 3100 to 0.129 to 108LaTeX: \frac{1}{12} 1 12 to 15 to 601 to 12Flag this Question Given that every fifteenth person in line will get a coupon for a free box of popcorn at the movies, what is the probability that you dont get a coupon when youre in line? Enter a fraction or round your answer to 4 decimal places, if necessary. which following statements are true? 11.5=11.05, 17.18 2.48 Assessment3/10Imagine you've used your own money to develop your business idea. Now you need morefunding to keep growing. Which financing method would be available to you at thisstage?A. Debt financingB. Angel investingC. Venture capitalD. Going public Poor, yeoman farmers who lived self-sufficiently and raised their own food, tended to purchasedvery little in the Market Economy.TrueFalse Find the volume of oxygen in a room with the dimensions of 144.78 cm x 198.12 cm x 144.78 The relationship between money earned and hours worked is linear joke and puetz the slope between 430 and 1290 that computes the slope between 4:30 and 10 75 how do the two slopes compare Find the missing number so that the equation has infinitely many solutions. -4x+20=blank x+20 Sam bought 3 boxes of chocolate online. Postage was $9 and the total cost was $45. How much was each box? which statement best describes what happens when you hold a cold drink in your hand? Please help!!Which statement best explains the text's use of dramatic irony?A. When Rich writes a fake name on the exam, he is not yet awarethat he will have to take the class again.B. When Yann Scheele turns in his test, the reader knows he will failor be accused of cheating,C. When Rich writes a fake name on the exam, the audience is awarethat Rich will fail Particle Physics this semester.D. When Yann Scheele turns in his test, the author portrays thecharacter as hardworking and confident. Help!!!!A student reads that some pea plants have violet flowers, while some have white flowers. Breeding a pea plant with violet flowers with a pea plant with white flowers produces offspring,some of which have violet flowers, while some have white flowers.How do the offspring express the gene for flower color from the parent plants?the random expression of one gene obtained from a single parent plantthe expression of one gene obtained from each parent plantthe expression of a gene that is inherited by any cell of the offspringthe random expression of a gene that is more numerous in the cells of the offspring Change the voice :(i) He should change his subject.(ii) What have you done ?(iii) Who taught you English ? The_,also known the true skin consists of nails,blood vessels,hair roots,sweat and sebaceous gland,and sensory mechanoreceptors. Please help me fast and i brainliest!! Two balls are released from the same height. Ball A is released on the surface of Earth, and ball B is released on the surface of the moon. The mass and net force on each ball are recorded in the table. Net Force (N) Mass (kg) Ball A 19.6 2 Ball B 9.6 6 Ball C 6.6 ? Which statement is correct about the mass of ball C, which is released on the surface of the moon from the same height? The mass of ball C is less than the masses of ball A and ball B. The mass of ball C cannot be determined from the information provided. The mass of ball C is greater than the masses of ball A and ball B. The mass of ball C is greater than the mass of ball A but less than the mass of ball B can someone give me the answers to this question What is -2/3 plus 1/5? Solve the equation vf=vi+at for a Graph the given function. State the domain and range. y=-116-9x+1